Can not figure out complexity of this recurrence - algorithm

I am refreshing on Master Theorem a bit and I am trying to figure out the running time of an algorithm that solves a problem of size n by recursively solving 2 subproblems of size n-1 and combine solutions in constant time.
So the formula is:
T(N) = 2T(N - 1) + O(1)
But I am not sure how can I formulate the condition of master theorem.
I mean we don't have T(N/b) so is b of the Master Theorem formula in this case b=N/(N-1)?
If yes since obviously a > b^k since k=0 and is O(N^z) where z=log2 with base of (N/N-1) how can I make sense out of this? Assuming I am right so far?

ah, enough with the hints. the solution is actually quite simple. z-transform both sides, group the terms, and then inverse z transform to get the solution.
first, look at the problem as
x[n] = a x[n-1] + c
apply z transform to both sides (there are some technicalities with respect to the ROC, but let's ignore that for now)
X(z) = (a X(z) / z) + (c z / (z-1))
solve for X(z) to get
X(z) = c z^2 / [(z - 1) * (z-a)]
now observe that this formula can be re-written as:
X(z) = r z / (z-1) + s z / (z-a)
where r = c/(1-a) and s = - a c / (1-a)
Furthermore, observe that
X(z) = P(z) + Q(z)
where P(z) = r z / (z-1) = r / (1 - (1/z)), and Q(z) = s z / (z-a) = s / (1 - a (1/z))
apply inverse z-transform to get that:
p[n] = r u[n]
and
q[n] = s exp(log(a)n) u[n]
where log denotes the natural log and u[n] is the unit (Heaviside) step function (i.e. u[n]=1 for n>=0 and u[n]=0 for n<0).
Finally, by linearity of z-transform:
x[n] = (r + s exp(log(a) n))u[n]
where r and s are as defined above.
so relabeling back to your original problem,
T(n) = a T(n-1) + c
then
T(n) = (c/(a-1))(-1+a exp(log(a) n))u[n]
where exp(x) = e^x, log(x) is the natural log of x, and u[n] is the unit step function.
What does this tell you?
Unless I made a mistake, T grows exponentially with n. This is effectively an exponentially increasing function under the reasonable assumption that a > 1. The exponent is govern by a (more specifically, the natural log of a).
One more simplification, note that exp(log(a) n) = exp(log(a))^n = a^n:
T(n) = (c/(a-1))(-1+a^(n+1))u[n]
so O(a^n) in big O notation.
And now here is the easy way:
put T(0) = 1
T(n) = a T(n-1) + c
T(1) = a * T(0) + c = a + c
T(2) = a * T(1) + c = a*a + a * c + c
T(3) = a * T(2) + c = a*a*a + a * a * c + a * c + c
....
note that this creates a pattern. specifically:
T(n) = sum(a^j c^(n-j), j=0,...,n)
put c = 1 gives
T(n) = sum(a^j, j=0,...,n)
this is geometric series, which evaluates to:
T(n) = (1-a^(n+1))/(1-a)
= (1/(1-a)) - (1/(1-a)) a^n
= (1/(a-1))(-1 + a^(n+1))
for n>=0.
Note that this formula is the same as given above for c=1 using the z-transform method. Again, O(a^n).

Don't even think about Master's Theorem. You can only use Masther's Theorem when you're given master's theorem when b > 1 from the general form T(n) = aT(n/b) + f(n).
Instead, think of it this way. You have a recursive call that decrements the size of input, n, by 1 at each recursive call. And at each recursive call, the cost is constant O(1). The input size will decrement until it reaches 1. Then you add up all the costs that you used to make the recursive calls.
How many are they? n. So this would take O(2^n).

Looks like you can't formulate this problem in terms of the Master Theorem.
A good start is to draw the recursion tree to understand the pattern, then prove it with the substitution method. You can also expand the formula a couple of times and see where it leads.
See also this question which solves 2 subproblems instead of a:
Time bound for recursive algorithm with constant combination time

May be you could think of it this way
when
n = 1, T(1) = 1
n = 2, T(2) = 2
n = 3, T(3) = 4
n = 4, T(4) = 8
n = 5, T(5) = 16
It is easy to see that this is a geometric series 1 + 2+ 4+ 8 + 16..., the sum of which is
first term (ratio^n - 1)/(ratio - 1). For this series it is
1 * (2^n - 1)/(2 - 1) = 2^n - 1.
The dominating term here is 2^n, therefore the function belongs to Theta(2^n). You could verify it by doing a lim(n->inf) [2^n / (2^n - 1)] = +ve constant.
Therefore the function belongs to Big Theta (2^n)

Related

How to determine Big O time complexity of the following recursion: f(n) = f(n-1) + f(n-3) + c

How can I solve this recursion :
f(n) = f(n-1) + f(n-3) + c.
where c is constant
f(1)=1
Please help me to solve this
We explicitly solve for the function. First, add c to both sides to get
f(n) + c = (f(n - 1) + c) + (f(n - 3) + c)
Define g(n) = f(n) + c. Then the recurrence for g is
g(n) = g(n - 1) + g(n - 3)
Note that the set of solutions to this equation forms a vector space of dimension 3, since g is determined by g(0), g(1), g(2). So it suffices to find 3 basis elements.
We try an Ansatz of g(n) = k^n for k nonzero. For such an Ansatz, we see that
k^n = k^(n - 1) + k^(n - 3)
In other words, we see that
k^3 = k^2 + 1
This equation has no rational roots, so its roots will be a bit nasty. It turns out that there are three roots, one real and two complex. The roots are approximately
x = 1.47
x = -0.233 +/- 0.793 i
So the three basic solutions are 1.47^n and (-0.233 +/- 0.793 i)^n (approximately).
You did not actually give enough information to find g, since we need to know g(0), g(1), and g(2). But for the basic solutions, it's easy to see that 1.47^n = O(1.47^n) and (-0.233 +/- 0.793 i)^n = O(1), since |(-0.233 +/- 0.793 i)| < 1.
Therefore, in almost all cases (except the ones where we can write g(n) = a (-0.233 + 0.793 i)^n + b (-0.233 - 0.793 i)^n), we have g(n)= Theta(1.47^n) and hence f(n) = g(n) - c = Theta(1.47^n).
Keep in mind that 1.47 is, of course, an approximation. The true value is the unique real solution of k^3 = k^2 + 1.

Analyzing Worst Case Performance of Quicksort by Substitution Method

I am trying to solve the reccurence of the quicksort algorithm by the substitution method:
I can not find any way to proof that this will lead to . What further steps do I have to make to make this work?
The worst case of quicksort is when you choose a pivot element which is the minimum or maximum element from the array, so that all remaining elements go to one side of the partition, and the other side of the partition is empty. In this case, the non-empty partition has a size of (n - 1), and it takes linear time (kn for some constant k > 0) to do the partitioning itself, so the recurrence relation is
T(n) = T(n - 1) + T(0) + kn
If we guess that T(n) = an² + bn + c for some constants a, b, c, then we can substitute:
an² + bn + c = [ a(n - 1)² + b(n - 1) + c ] + [ c ] + kn
where the two square-bracketed terms are T(n - 1) and T(0) respectively. By expanding the brackets and equating coefficients, we get
an² = an²
bn = -2an + bn + kn
c = a - b + 2c
It follows that there is a family of solutions, parameterised by c = T(0), where a = k/2 and b = k/2 + c. This family of solutions can be written exactly as
T(n) = (k/2) n² + (k/2 + c) n + c
which is not just O(n²), but Ө(n²), meaning the running time is a quadratic function, not merely bounded above by a quadratic function. Note that the actual value of c doesn't change the asymptotic behaviour of the function, so long as k > 0 (i.e. the partitioning step does take a positive amount of time).
I have found an answer to my question, the continuation of the previous equation is,
This is true if

Time complexity of the program using recurrence equation

I want to find out the time complexity of the program using recurrence equations.
That is ..
int f(int x)
{
if(x<1) return 1;
else return f(x-1)+g(x);
}
int g(int x)
{
if(x<2) return 1;
else return f(x-1)+g(x/2);
}
I write its recurrence equation and tried to solve it but it keep on getting complex
T(n) =T(n-1)+g(n)+c
=T(n-2)+g(n-1)+g(n)+c+c
=T(n-3)+g(n-2)+g(n-1)+g(n)+c+c+c
=T(n-4)+g(n-3)+g(n-2)+g(n-1)+g(n)+c+c+c+c
……………………….
……………………..
Kth time …..
=kc+g(n)+g(n-1)+g(n-3)+g(n-4).. .. . … +T(n-k)
Let at kth time input become 1
Then n-k=1
K=n-1
Now i end up with this..
T(n)= (n-1)c+g(n)+g(n-1)+g(n-2)+g(n-3)+….. .. g(1)
I ‘m not able to solve it further.
Any way if we count the number of function calls in this program , it can be easily seen that time complexity is exponential but I want proof it using recurrence . how can it be done ?
Explanation in Anwer 1, looks correct , similar work I did.
The most difficult task in this code is to write its recursion equation. I have drawn another diagram , I identified some patterns , I think we can get some help form this diagram what could be the possible recurrence equation.
And I came up with this equation , not sure if it is right ??? Please help.
T(n) = 2*T(n-1) + c * logn
Ok, I think I have been able to prove that f(x) = Theta(2^x) (note that the time complexity is the same). This also proves that g(x) = Theta(2^x) as f(x) > g(x) > f(x-1).
First as everyone noted, it is easy to prove that f(x) = Omega(2^x).
Now we have the relation that f(x) <= 2 f(x-1) + f(x/2) (since f(x) > g(x))
We will show that, for sufficiently large x, there is some constant K > 0 such that
f(x) <= K*H(x), where H(x) = (2 + 1/x)^x
This implies that f(x) = Theta(2^x), as H(x) = Theta(2^x), which itself follows from the fact that H(x)/2^x -> sqrt(e) as x-> infinity (wolfram alpha link of the limit).
Now (warning: heavier math, perhap cs.stackexchange or math.stackexchange is better suited)
according to wolfram alpha (click the link and see series expansion near x = infinity),
H(x) = exp(x ln(2) + 1/2 + O(1/x))
And again, according to wolfram alpha (click the link (different from above) and see the series expansion for x = infinity), we have that
H(x) - 2H(x-1) = [1/2x + O(1/x^2)]exp(x ln(2) + 1/2 + O(1/x))
and so
[H(x) - 2H(x-1)]/H(x/2) -> infinity as x -> infinity
Thus, for sufficiently large x (say x > L) we have the inequality
H(x) >= 2H(x-1) + H(x/2)
Now there is some K (dependent only on L (for instance K = f(2L))) such that
f(x) <= K*H(x) for all x <= 2L
Now we proceed by (strong) induction (you can revert to natural numbers if you want to)
f(x+1) <= 2f(x) + f((x+1)/2)
By induction, the right side is
<= 2*K*H(x) + K*H((x+1)/2)
And we proved earlier that
2*H(x) + H((x+1)/2) <= H(x+1)
Thus f(x+1) <= K * H(x+1)
Using memoisation, both functions can easily be computed in O(n) time. But the program takes at least O(2^n) time, and thus is a very inefficient way of computing f(n) and g(n)
To prove that the program takes at most O(2+epsilon)^n time for any epsilon > 0:
Let F(n) and G(n) be the number of function calls that are made in evaluating f(n) and g(n), respectively. Clearly (counting the addition as 1 function call):
F(0) = 1; F(n) = F(n-1) + G(n) + 1
G(1) = 1; G(n) = F(n-1) + G(n/2) + 1
Then one can prove:
F and G are monotonic
F > G
Define H(1) = 2; H(n) = 2 * H(n-1) + H(n/2) + 1
clearly, H > F
for all n, H(n) > 2 * H(n-1)
hence H(n/2) / H(n-1) -> 0 for sufficiently large n
hence H(n) < (2 + epsilon) * H(n-1) for all epsilon > 0 and sufficiently large n
hence H in O((2 + epsilon)^n) for any epsilon > 0
(Edit: originally I concluded here that the upper bound is O(2^n). That is incorrect,as nhahtdh pointed out, but see below)
so this is the best I can prove.... Because G < F < H they are also in O((2 + epsilon)^n) for any epsilon > 0
Postscript (after seeing Mr Knoothes solution): Because i.m.h.o a good mathematical proof gives insight, rather than lots of formulas, and SO exists for all those future generations (hi gals!):
For many algorithms, calculating f(n+1) involves twice (thrice,..) the amount of work for f(n), plus something more. If this something more becomes relatively less with increasing n (which is often the case) using a fixed epsilon like above is not optimal.
Replacing the epsilon above by some decreasing function ε(n) of n will in many cases (if ε decreases fast enough, say ε(n)=1/n) yield an upper bound O((2 + ε(n))^n ) = O(2^n)
Let f(0)=0 and g(0)=0
From the function we have,
f(x) = f(x - 1) + g(x)
g(x) = f(x - 1) + g(x/2)
Substituting g(x) in f(x) we get,
f(x) = f(x-1) + f(x -1) + g(x/2)
∴f(x) = 2f(x-1) + g(x/2)
Expanding this we get,
f(x) = 2f(x-1)+f(x/2-1)+f(x/4-1)+ ... + f(1)
Let s(x) be a function defined as follows,
s(x) = 2s(x-1)
Now clearly f(x)=Ω(s(x)).
The complexity of s(x) is O(2x).
Therefore function f(x)=Ω(2x).
I think is clear to see that f(n) > 2n, because f(n) > h(n) = 2h(n-1) = 2n.
Now I claim that for every n, there is an ε such that:
f(n) < (2+ε)n, to see this, let do it by induction, but to make it more sensible at first I'll use ε = 1, to show f(n) <= 3n, then I'll extend it.
We will use strong induction, suppose for every m < n, f(m) < 3m then we have:
f(n) = 2[f(n-1) + f(n/2 -1) + f(n/4 -1)+ ... +f(1-1)]
but for this part:
A = f(n/2 -1) + f(n/4 -1)+ ... +f(1-1)
we have:
f(n/2) = 2[f(n/2 -1) + f(n/4 -1)+ ... +f(1-1]) ==>
A <= f(n/2) [1]
So we can rewrite f(n):
f(n) = 2f(n-1) + A < 2f(n-1) +f(n/2),
Now let back to our claim:
f(n) < 2*3^(n-1) + 2*3^(n/2)==>
f(n) < 2*3^(n-1) + 3^(n-1) ==>
f(n) < 3^n. [2]
By [2], proof of f(n)&in;O(3n) is completed.
But If you want to extend this to the format of (2+ε)n, just use 1 to replace the inequality, then we will have
for ε > 1/(2+ε)n/2-1 → f(n) < (2+ε)n.[3]
Also by [3] you can say that for every n there is an ε such that f(n) < (2+ε)n actually there is constant ε such that for n > n0, f(n)&in;O((2+ε)n). [4]
Now we can use wolfarmalpha like #Knoothe, by setting ε=1/n, then we will have:
f(n) < (2+1/n)n which results on f(n) < e*2n, and by our simple lower bound at start we have: f(n)&in; Θ(2^n).[5]
P.S: I didn't calculate epsilon exactly, but you can do it with pen and paper simply, I think this epsilon is not correct, but is easy to find it, and if is hard tell me is hard, and I'll write it.

The Recurrence W(n)= 2W(floor(n/2)) + 3

I have this recurrence:
W(n)= 2W(floor(n/2)) + 3
W(2)=2
My try is as follow:
the tree is like this:
W(n) = 2W(floor(n/2)) + 3
W(n/2) = 2W(floor(n/4)) + 3
W(n/4) = 2W(floor(n/8)) + 3
...
the hight of the tree : I assume its lgn because the tree has 2 branches at every expanding process, not sure though :S
the cost of the last level : 2^lgn * W(2) = 2n
the cost of all levels until level h-1 : 3 * sigma from 0 to lgn-1 of (2^i), which is a geometric series = 3 (n-1)
So, T(n) = 5n - 3 which belong to Theta(n)
my question is: Is that right?
I don't think it's exactly 5n-3 except n is 2t, but your theta is right if you look at Master Theorem, there is no need to calculate it (but its good for startup):
assume you have:
T(n) = aT(n/b) + f(n), where a>=1, b>1 then:
if f(n) = nlogba-eps for any eps > 0 then T(n) = nlogba like your case, in which a=b=2, f(n) = O(1).
f(n) = Theta(nlogba * logkn) then T(n)=Theta(nlogba * logk+1n).
Otherwise is Theta(f(n)). (see detail of constraint in this case in CLRS or wiki, ...)
for detail see wiki.
Well, if you calculate W(4), you find W(4) = 2*W(2) + 3 = 2*2 + 3 = 7, but 5*4 - 3 = 17, so your result for T(n) is not correct. It is close, though, there's just a minor slip in your reasoning (or possibly in a certain other place).
Edit: To be specific, your calculation would work if W(1) was given, but it's W(2) in the question. Either the latter is a typo or you're off by one with the height. (and of course, what Saeed Amiri said.)

Can someone help solve this recurrence relation? [closed]

Closed. This question does not meet Stack Overflow guidelines. It is not currently accepting answers.
This question does not appear to be about programming within the scope defined in the help center.
Closed 8 years ago.
Improve this question
T(n) = 2T(n/2) + 0(1)
T(n) = T(sqrt(n)) + 0(1)
In the first one I use substitution method for n, logn, etc; all gave me wrong answers.
Recurrence trees: I don't know if I can apply as the root will be a constant.
Can some one help?
Let's look at the first one. First of all, you need to know T(base case). You mentioned that it's a constant, but when you do the problem it's important that you write it down. Usually it's something like T(1) = 1. I'll use that, but you can generalize to whatever it is.
Next, find out how many times you recur (that is, the height of the recursion tree). n is your problem size, so how many times can we repeatedly divide n by 2? Mathematically speaking, what's i when n/(2^i) = 1? Figure it out, hold onto it for later.
Next, do a few substitutions, until you start to notice a pattern.
T(n) = 2(2(2T(n/2*2*2) + θ(1)) + θ(1)) + θ(1)
Ok, the pattern is that we multiply T() by 2 a bunch of times, and divide n by 2 a bunch of times. How many times? i times.
T(n) = (2^i)*T(n/(2^i)) + ...
For the big-θ terms at the end, we use a cute trick. Look above where we have a few substitutions, and ignore the T() part. We want the sum of the θ terms. Notice that they add up to (1 + 2 + 4 + ... + 2^i) * θ(1). Can you find a closed form for 1 + 2 + 4 + ... + 2^i? I'll give you that one; it's (2^i - 1). It's a good one to just memorize, but here's how you'd figure it out.
Anyway, all in all we get
T(n) = (2^i) * T(n/(2^i)) + (2^i - 1) * θ(1)
If you solved for i earlier, then you know that i = log_2(n). Plug that in, do some algebra, and you get down to
T(n) = n*T(1) + (n - 1)*θ(1). T(1) = 1. So T(n) = n + (n - 1)*θ(1). Which is n times a constant, plus a constant, plus n. We drop lower order terms and constants, so it's θ(n).
Prasoon Saurav is right about using the master method, but it's important that you know what the recurrence relation is saying. The things to ask are, how much work do I do at each step, and what is the number of steps for an input of size n?
Use Master Theorem to solve such recurrence relations.
Let a be an integer greater than or equal to 1 and b be a real number greater than
1. Let c be a positive real number and
d a nonnegative real number. Given a recurrence of the form
T (n) = a T(n/b) + nc .. if n > 1
T(n) = d .. if n = 1
then for n a power of b,
if logb a < c, T (n) = Θ(nc),
if logb a = c, T (n) = Θ(nc log n),
if logb a > c, T (n) = Θ(nlogb a).
1) T(n) = 2T(n/2) + 0(1)
In this case
a = b = 2;
logb a = 1; c = 0 (since nc =1 => c= 0)
So Case (3) is applicable. So T(n) = Θ(n) :)
2) T(n) = T(sqrt(n)) + 0(1)
Let m = log2 n;
=> T(2m) = T( 2m / 2 ) + 0(1)
Now renaming K(m) = T(2m) => K(m) = K(m/2) + 0(1)
Apply Case (2).
For part 1, you can use Master Theorem as #Prasoon Saurav suggested.
For part 2, just expand the recurrence:
T(n) = T(n ^ 1/2) + O(1) // sqrt(n) = n ^ 1/2
= T(n ^ 1/4) + O(1) + O(1) // sqrt(sqrt(n)) = n ^ 1/4
etc.
The series will continue to k terms until n ^ 1/(2^k) <= 1, i.e. 2^k = log n or k = log log n. That gives T(n) = k * O(1) = O(log log n).
Let's look at the first recurrence, T(n) = 2T(n/2) + 1. The n/2 is our clue here: each nested term's parameter is half that of its parent. Therefore, if we start with n = 2^k then we will have k terms in our expansion, each adding 1 to the total, before we hit our base case, T(0). Hence, assuming T(0) = 1, we can say T(2^k) = k + 1. Now, since n = 2^k we must have k = log_2(n). Therefore T(n) = log_2(n) + 1.
We can apply the same trick to your second recurrence, T(n) = T(n^0.5) + 1. If we start with n = 2^2^k we will have k terms in our expansion, each adding 1 to the total. Assuming T(0) = 1, we must have T(2^2^k) = k + 1. Since n = 2^2^k we must have k = log_2(log_2(n)), hence T(n) = log_2(log_2(n)) + 1.
Recurrence relations and recursive functions as well should be solved by starting at f(1). In case 1, T(1) = 1; T(2) = 3; T(4) = 7; T(8) = 15; It's clear that T(n) = 2 * n -1, which in O notation is O(n).
In second case T(1) = 1; T(2) = 2; T(4) = 3; T(16) = 4; T(256) = 5; T(256 * 256) =6; It will take little time to find out that T(n) = log(log(n)) + 1 where log is in base 2. Clearly this is O(log(log(n)) relation.
Most of the time the best way to deal with recurrence is to draw the recurrence tree and carefully handle the base case.
However here I will give you slight hint to solve using substitution method.
In recurrence first try substitution n = 2^k
In recurrence second try substitution n = 2^2^k

Resources